exam 3

Pataasin ang iyong marka sa homework at exams ngayon gamit ang Quizwiz!

A client has been diagnosed with alcohol-induced liver disease. He admits to the nurse, "I know what the lungs do, and I know what the heart does, but honestly, I have no idea what the liver does in the body." The nurse should tell the client that the liver:

metabolizes most components of food and also cleans the blood of bacteria and drugs.

A 45-year-old Caucasian woman is being treated for ovarian cancer. Her treatment involves the chemotherapy agent cisplatin. The nurse should monitor the client for signs and symptoms of:

nephrotoxic acute tubular necrosis.

A client with chronic kidney disease has developed cardiac calcification. On admission the priority assessment would be for the nurse to:

place on a heart monitor to watch for arrhythmias.

A client with a diagnosis of end-stage renal disease received a kidney transplant 2 years ago that was deemed a success. During the most recent follow-up appointment, the nurse should prioritize the client for referral based on which statement?

"I'm feeling a bit under the weather these days and I'm a bit feverish."

A client who suffers from spastic bladder has been catheterized to promote bladder emptying. Which of the following medications should the nurse plan on the physician ordering to also treat this problem?

Anticholinergic medication

Which of the following types of pharmacological therapy does the nurse anticipate administering to a patient for treatment of a spastic bladder in order to decrease bladder hyperactivity?

Anticholinergic medications

Which methods are most commonly used to treat detrusor-sphincter dyssynergia?

Anticholinergic medications and urinary catheterization

A patient has just been diagnosed with acute glomerulonephritis. Which question should the nurse ask this client in attempting to establish a cause?

"Have you had any type of infection within the last 2 weeks?"

A 42 year-old male has been diagnosed with renal failure secondary to diabetes mellitus and is scheduled to begin dialysis soon. Which of the following statements by the client reflects an accurate understanding of the process of hemodialysis?

"I won't be able to go about my normal routine during treatment."

Many drugs are metabolized and detoxified in the liver. Most drug metabolizing occurs in the central zones of the liver. What condition is caused by these drug-metabolizing actions?

Centrilobular necrosis

A nurse is completing a history and physical assessment on a patient admitted with ulcerative colitis. Which of the following symptoms would the nurse anticipate? Select all that apply.

Diarrhea Rectal bleeding

A client who is diagnosed with Zollinger-Ellison syndrome will exhibit which symptoms?

Diarrhea with fat deposits

A patient with chronic kidney disease (CKD) will begin hemodialysis. The nurse will recommend which of the following diets?

Diet low in proteins but including eggs and lean meat

The nurse is instructing a patient with advanced kidney disease (AKD) about a dietary regimen. Which of the following restrictions should the nurse be sure to include in the treatment plan to decrease the progress of renal impairment in people with AKD?

Dietary protein

A 1-year-old baby boy with renal dysplasia risks end-stage renal disease unless intervention occurs. Which of the following treatment options is his care team most likely to reject?

Dietary restriction plus erythropoietin

A patient complains of having bouts of diarrhea. The patient wants to know which antidiarrheal would be best to use. Which of the following antidiarrheals does the nurse know will help stimulate water and electrolyte absorption?

Diphenoxylate (Lomotil)

A client with chronic kidney disease (CKD) has developed asterixis. The nurse knows that asterixis is which of the following?

Dorsiflexion of hands and feet

A client newly diagnosed with pancreatic cancer is admitted to begin treatment. Which pain discriptors can be associated with adenocarcinomas of the pancreas?

Dull epigastric pain accompanied by back pain, worse when lying flat and relieved by sitting forward.

Which of the following pain descriptions would lead the nurse to suspect the client is experiencing ureteral colic?

Excruciating pain in the flank and upper outer quadrant of abdomen that radiates to bladder area.

Which urinary structure helps to stop micturition when it is occurring and maintains continence under high bladder pressure?

External sphincter

What is a common cause of spastic bladder dysfunction?

External sphincter spasticity

Which of the following is a nonsurgical method of treatment for renal calculi (kidney stones)?

Extracorporeal shock wave lithotripsy (ESWL)

The nurse is teaching her patient with hepatobiliary disease about her diet. She tells her that she may have steatorrhea, which is the malabsorption of which dietary component?

Fat

The nurse recognizes which of these complications results from liver dysfunction with decrease in production of bile salts?

Fat soluble vitamin deficiencies

The nurse knows that uremia or "urine in the blood" is often used to describe the clinical manifestations of chronic kidney disease (CKD). Which of the following are early signs and symptoms of uremia? Select all that apply.

Fatigue Nausea Apathy Weakness

The nurse is caring for a client with hepatitis and jaundice. The nurse recognizes that without sufficient circulating bile salts the client will have intolerance to which of these ingested substances?

Fats

Good hand-washing techniques are important in healthcare. The nurse knows that bacterial infections can be prevented by good hand-washing techniques. Which of the following routes of transmission is most common for Clostridium difficile?

Fecal-oral transmission

The nurse at a clinic reviews client histories to determine if the hepatitis B vaccine should be given. Which clients should receive the vaccine? Select all that apply.

Female surgical scrub technician Male with HIV infection Female newborn whose mother has HBV Male soldier posted to the Middle East

The patient who has been admitted with a problem with his bladder has a postvoid residual (PVR) of 250 ml. The nurse understands that this indicates which of the following?

Inadequate bladder emptying

Which of the following assessments indicates to the nurse that a patient may have a spastic bladder dysfunction?

Incontinence

A client reports that she frequently suffers from UTIs after engaging in sexual intercourse. Which would be the best information for the nurse to provide?

Increase fluid intake before intercourse

The nurse is caring for a client alcoholism who now presents with gastrointestinal bleeding. What alteration does the nurse anticipate related to the bleeding?

Increase in ammonia level

The nurse is assessing a patient who has a unilateral obstruction of the urinary tract. The nurse anticipates the patient may develop:

Increase in blood pressure

A patient diagnosed with flaccid neurogenic bladder is taught to apply pressure with the hand above the symphysis pubis while in the sitting position. What is the expected outcome of this technique?

Increase in intravesical pressure

Manifestations of polycystic kidney disease include which of the following?

Increase in kidney size bilaterally

A nurse advises a client with recurring UTIs to drink large amounts of water. What normal protective action is the nurse telling the client to utilize?

Increase washout of urine

The nurse is planning care for a patient with a urinary tract obstruction. The nurse includes assessment for which of the following possible complications?

Increased blood pressure

Unilateral obstruction of the urinary tract may result in renin secretion, thereby leading to which of the following?

Increased blood pressure

Albuminuria is an important indicator of nephron damage. The nurse recognizes that which of the following statements regarding albuminuria are correct? Select all that apply.

Increased excretion of albumin indicates CKD. • Urine dipstick can detect albuminuria. • Albumin-to-creatinine ratio can confirm albuminuria.

Which of the following data would a clinician consider as most indicative of acute renal failure?

Increased nitrogenous waste levels; decreased glomerular filtration rate (GFR).

The nurse is reviewing lab results of a client who has liver failure. The nurse determines that the client is at an increased risk for bleeding when the results include:

Increased prothrombin time

When caring for the client with hepatic failure, the nurse recognizes which of these problems places the client at increased risk for bleeding?

Increased prothrombin time

The nurse is reviewing the lab results of a client diagnosed with cholestasis. The nurse would anticipate:

Increased serum alkaline phosphate

A patient is to receive a radiocontrast media as part of a diagnostic scan. Which of the following is intended to reduce the nephrotoxic effects of the radiocontrast media?

Increasing the normal saline intravenous infusion rate to 125 mL/hour

Magnesium renal stones are associated with which of the following conditions?

Infection

Which one of the following is the usual cause of acute pyelonephritis?

Infection

A patient is admitted to the medical surgical unit with a history of inflammatory bowel disease. The nurse knows that the clinical manifestations of both Crohn's disease and ulcerative colitis are the result of activation of which of the following cells?

Inflammatory

A 22 year-old student has developed a fever and diarrhea while on a backpacking trip in Southeast Asia. His oral temperature is 101.40 F. The diarrhea is bloody, frequent, and small in volume. These clinical manifestations are sufficiently distressing that he/she is visiting a local medical clinic in the area. Which of the following diagnoses best characterizes this health problem?

Inflammatory diarrhea

Crohn's disease is treated by several measures. Treatment with corticosteroids focuses on which of the following?

Inflammatory suppression

Crohn disease is a recurrent inflammatory disease that can affect any area of the bowel. Characteristic of Crohn disease are granulomatous lesions that are sharply demarcated from the surrounding tissue. The nurse caring for a client with newly diagnosed Crohn disease would know to include what in the teaching?

Information on sulfasalazine including dosage, route, frequency, and side effects of the drug

The community health nurse is teaching about prevention of hepatitis A. Which of these groups does the nurse suggest will benefit from this vaccine?

Those traveling to third world countries

A client with a neurogenic bladder has a lesion at the level of sacral reflexes/peripheral nerves that innervate the bladder. The nurse anticipates the client will experience:

flaccid bladder dysfunciton

A patient diagnosed with Goodpasture's syndrome would require which of the therapies to remove proteins and autoantibodies from the system?

Plasmapheresis

Which one of the following would the nurse see as being liable to cause the most serious long-term problems?

Polycystic kidney disease

The nurse knows that one of the earliest manifestations of acute renal failure (ARF) is which of the following?

Polyuria

A client has a tubulointerstitial disorder and is no able to concentrate urine. The nurse will likely assess which manifestations associated with this disorder? Select all that apply.

Polyuria Nocturia

A client is diagnosed with chronic kidney disease (CKD). The nurse will monitor this client for which of the following Select all that apply.

Polyuria • Hyperkalemia • Hypocalcemia • Hyponatremia

A patient has an obstructive urine outflow related to benign prostatic hyperplasia. Due to the inability to excrete adequate amounts of urine, which of the following types of renal failure should the nurse closely monitor for?

Postrenal failure

Congenital disorders of the kidneys are fairly common, occurring in approximately 1:1000 live births. What is the result to the newborn when bilateral renal dysplasia occurs? (Select all that apply.)

Potter facies Oligohydramnios Pulmonary hypoplasia Renal failure

A 35-year-old client is diagnosed with acute kidney injury (AKI) and is started on hemodialysis. The client is concerned with the diagnosis and wants to know what to expect in the progression of this disorder. Which statement best addresses the client's concern?

"Acute kidney injury is abrupt in onset and often reversible if recognized early and treated appropriately."

A client with a history of cancer that metastasized to the liver has arrived at the outpatient clinic to have a paracentesis performed. The physician anticipates that the client will have more than 5 L of fluid removed. The physician has prescribed intravenous albumin following the procedure. The client asks why she needs, "more fluids in my vein?" The nurse responds:

"Albumin is a volume expander. Since a lot of fluid was removed, you have a decrease in your vascular volume, so without this albumin, your kidneys will try to reabsorb and hold onto water."

The nursing instructor who is teaching about disorders of the lower urinary tract realizes a need for further instruction when one of the students makes which of the following statements?

"Alterations in bladder function can only occur when there is incontinence."

A 24 year-old man is currently in a rehabilitation facility following a spinal cord injury at level T2. He is discussing his long term options for continence management. Which of the following statements by the client demonstrates he has a clear understanding of the issue?

"An indwelling catheter certainly would work well, but it comes with a number of risks and possible complications."

A young mother asks,"Why can my 3-year-old daughter have a bowel movement on the toilet but she wets her pants?" Which of the following is the nurse's best response?

"Bowel control occurs earlier than bladder control."

The nursing instructor who is teaching about incontinence in the elderly recognizes a need for further instruction when one of the students makes which of the following statements?

"Frequency is not a major problem for the elderly."

The nurse is caring for a female client with cholelithiasis. When teaching the client about the disease, the nurse includes which of these points?

"Gallstones have developed, which are typically composed of cholesterol."

The nurse is caring for a client with a diagnosis of post-hepatic obstruction. Which of these questions will assist the nurse in determining the underlying cause of the obstruction?

"Have you been told you have had gallstones?"

The nurse is teaching the client about dietary factors that increase risks for colon cancer. The most important information for the nurse to include would be: Select all that apply.

"Increasing your daily amount of sugar intake does influence your risk." "Decrease your daily intake of vitamins A and E."

A diabetes education nurse is teaching a group of recently diagnosed diabetics about the potential genitourinary complications of diabetes and the consequent importance of vigilant blood glucose control. Which of the following teaching points best conveys an aspect of bladder dysfunction and diabetes mellitus?

"It's important for you to empty your bladder frequently because diabetes carries risks of kidney damage that can be exacerbated by incomplete bladder emptying."

A 63 year-old woman has visited a physician because she has been intermittently passing blood-tinged urine over the last several weeks, and cytology has confirmed a diagnosis of invasive bladder cancer. Which of the following statements by the physician is most accurate?

"It's likely that you'll need surgery; possibly a procedure called a cystectomy."

A nurse educator is performing client education with a 51 year-old man who has been recently diagnosed with chronic kidney disease. Which of the following statements by the client would the nurse most likely want to correct or clarify?

"My kidney problems increase my chance of developing high blood pressure or diabetes."

The client has just been diagnosed with bladder cancer and asks the nurse what causes it. Which of the following would be the nurse's best response to the client?

"The cause is unknown."

A patient has a spastic bladder. Which of the following is of most concern to the nurse?

"The patient has an elevated temperature."

A male patient has just been diagnosed with esophageal cancer. He knew he was losing weight and fatigued most days, but he just attributed it to aging and working. The physician recommends chemotherapy and irradiation. However, the cancer has already metastasized. The patient asks the nurse what he can expect if he agrees to the treatments. The nurse responds:

"The therapies may shrink the cancer in an effort to increase survival of the cancer."

Following several days of intermittent upper right quadrant pain, a 29 year-old obese, Native American woman has been diagnosed with cholelithiasis. The nurse at the clinic has taught the client about the pathophysiology and contributing factors to her health problem, as well as some of the likely treatment options. Which of the following statements by the client demonstrates a sound understanding of her diagnosis?

"This explains why my skin was yellow-tinged lately and why I had those pains that spread to my upper back and right shoulder."

A patient has a postvoid residual (PVR) volume of 40 mL. Which of the following information would the nurse teach the patient?

"This is a normal value."

A patient has a postvoid residual (PVR) volume of 250 mL. Which of the following information would the nurse tell the patient?

"This value indicates you are having difficulty emptying your bladder."

The clinical nurse educator on a nephrology unit of a large, urban hospital is orientating recent nursing graduates to the unit. Which of the following teaching points about acute tubular necrosis (ATN) should the educator include in the orientation session?

"Trauma, burns and major surgery are common precursors to ATN."

A 60 year-old man has been diagnosed with renal calculi after repeated episodes of excruciating flank pain in recent weeks. The man states that, "I don't know how this could happen to me, since I'm so careful about eating a healthy diet." What is the most appropriate response to the man's statement?

"What you eat can influence your risk of stone formation, but many other factors like hormones and your metabolism are involved."

A 51 year-old male professional is in the habit of consuming 6 to 8 rum and cokes each evening after work. He assures the nurse practitioner who is performing his regular physical exam that his drinking is under control and does not have negative implications for his work or family life. How could the nurse best respond to the client's statement?

"When your body has to regularly break down that much alcohol, your blood and the functional cells in your liver accumulate a lot of potentially damaging toxic byproducts."

A 71 year-old male has been recently diagnosed with a stage III tumor of colorectal cancer, and is attempting to increase his knowledge base of his diagnosis. Which of the following statements about colorectal cancer demonstrates a sound understanding of the disease?

"While diet is thought to play a role in the development of colorectal cancer, the ultimate causes are largely unknown."

A client experienced asymptomatic UTIs while pregnant. The client asks the nurse if this places her at any risk for complications. Which would be the best response by the nurse?

"You are at risk for developing acute pyelonephritis."

A client has been diagnosed with having calcium oxalate kidney stones following intravenous pyelography. Which of the following teaching points about the treatment of the health problem are justifiable? Select all that apply:

"You may need to cut out cocoa, chocolate and some nuts from your diet." "Extracorporeal shock-wave lithotripsy treatment may be used to fragment larger stones."

A client has undergone a kidney transplant and voices concerns about organ rejection to the nurse. The most appropriate response by the nurse would be:

"You will be given medication to decrease your immune systems from attacking your new kidney."

A mother asks, "why can't my 1 year old go to the bathroom by himself?" Which of the following is the nurse's best response?

"Your child is too young to begin toilet training."

A family physician is providing care for a 61 year-old obese male who has a history of diabetes and hypertension. Blood work has indicated that the man has a GFR of 51 mL/min with elevated serum creatinine levels. Which of the following statements will the physician most likely provide to the client in light of these results?

"Your chronic kidney disease has likely been caused by your diabetes and high blood pressure."

Following a history of gastric pain and an endoscopy, a client has been diagnosed with a duodenal peptic ulcer. Which of the following teaching points should his caregiver provide?

"Your family history, your smoking history and NSAID use may all have contributed to this problem."

A 34 year-old man has been taking up to 2400 mg of ibuprofen per day following a motor vehicle several months ago and consequent chronic pain. He has recently been diagnosed with chronic analgesic nephritis as a result of his high analgesic intake. The man is surprised at the diagnosis, stating, "I thought that taking too many drugs hurt your liver if anything, not your kidneys." What is the most appropriate response to the man's statement?

"Your kidneys are vulnerable to damage because of how much blood flows through them and the fact that they break down many drugs."

A frantic mother brings her young child into the emergency department. She states that during the evening bath, she noticed a large mass in her child's abdomen. After diagnostic testing, the pediatrician tells the parents that their child has Wilms tumor, Stage IV. After the doctor leaves the room, the parents ask the nurse, "What does this mean?" The nurse will respond, "Your child ('s):

"has cancer of the kidney that has spread most likely to their lungs."

Which of the following individuals are displaying identified risk factors for the development of lower urinary tract obstruction? Select all that apply.

68 year-old man who has been diagnosed with benign prostatic hyperplasia (BPH). 30 year-old woman who has been diagnosed with gonorrhea. 74 year-old woman who has developed a lower bowel obstruction following several weeks of chronic constipation. 20 year-old man who has spina bifida and consequent impaired mobility.

The nurse knows that a patient with which glomerular filtration rate (GFR) would be classified as "decreased GFR"?

70 mL/min

A client informs the nurse that he feels as if his bladder is getting full. The nurse understands that a client first experiences the sensation of bladder fullness when the bladder contains which of the following?

100-200 ml of urine

The health care provider is comparing results of a client's recent GFR measurement. Which of the following results would be interpreted as normal?

120 to 130 mL/minute

The nurse is teaching a patient with a spinal cord injury about self-catheterization. Which of the following measurements is important to include about the amount of urine that should be allowed to collect in the bladder between catheterizations?

300-400 mL

A patient will sense fullness of the bladder when the bladder contains what amount of urine?

400mL

A client is suspected to have chronic kidney disease (CKD). The nurse will use which glomerular filtration rate (GFR) to aid in this diagnosis?

60 ml/min/1.73 square meters or less for three months

Which of the following clients would be considered to have a significant risk of developing the prerenal form of acute renal failure? Select all that apply.

79 year-old male with diagnoses of poorly controlled diabetes mellitus and heart failure. 22 year-old male who has lost large amounts of blood following a workplace injury. 80 year-old female who has been admitted for the treatment of dehydration and malnutrition.

What is considered the normal amount of serum bilirubin found in the blood?

<1.5mg/dL

Which client on a hospital medical unit is most clearly demonstrating the signs and symptoms of liver failure?

A 44-year-old man with low hemoglobin levels, low platelet levels, and spider angiomas present

The nurse is reviewing the medical history of four clients. The nurse determines that the client at greatest risk for developing peptic ulcer disease is:

A client with a prior diagnosis of Helicobacter pylori and refused treatment

Which of the following clients is most clearly displaying the signs and symptoms of irritable bowel disease (IBD)?

A 51 year-old male who states that his stomach pain is in his lower abdomen, "comes and goes" and "feels more like a cramp than a dull ache."

Select the client at greatest risk for developing colorectal cancer.

A 64-year-old female whose mother had colorectal cancer

Select the client who is at the greatest risk of developing bladder cancer.

A 65-year-old white male with a history of bladder stones

A geriatric nurse is caring for several clients. Which alterations in clients' health should the nurse attribute to age-related physiologic changes?

A 78-year-old woman's GFR has been steadily declining over several years.

A patient has prerenal failure. The nurse knows that this type of failure is characterized by which relationship of blood urea nitrogen (BUN) to serum creatinine levels?

A BUN to creatinine level ratio of 20:1

The nurse is assessing a client who has just developed peritonitis. The nurse would expect the client to manifest:

A board-like abdomen

The nurse has been providing dietary teaching to a client diagnosed with irritable bowel syndrome. The nurse determines that the teaching was effective when the client selects:

A bran muffin, fruit, and orange juice.

The nurse on a geriatric unit is assessing four clients. Which client is most likely to exhibit pyuria and high counts of urine colony-forming units?

A client who has urinary catheter in place due to confusion

Which of the following clients' diagnostic bloodwork is most suggestive of chronic kidney disease (CKD)?

A client with low vitamin D levels; low calcitrol levels and elevated parathyroid hormone (PTH) levels

The nurse is educating a patient with chronic kidney disease (CKD). What is the recommended daily fluid intake for this patient?

A daily fluid intake of 500 to 800 mL/day to maintain hydration

The nurse recognizes hepatic encephalopathy may be aggravated by which of these situations?

A high protein meal

The nurse is evaluating patient risk for the development of overactive bladder/urge incontinence and determines that the patient at highest risk is which of the following?

A patient with diabetes mellitus

The nurse recognizes the most common cause of acute postinfectious glomerulonephritis as:

A streptococcal infection 7 to 12 days prior to onset

In which client would the nurse recognize the symptoms of a lesion in the micturition centre of the sacral cord?

A trauma client whose bladder ultrasound reveals 1,140 mL of urine

Four weeks after returning from a tropical vacation, a 40 year-old man has presented to the emergency department with malaise, nausea and yellow sclera. Serology has confirmed a diagnosis of hepatitis A (HAV). What teaching is most appropriate for this client?

A vaccine before your trip would have prevented this, but your body will rid itself of the virus in time.

A client has been diagnosed with systemic lupus erythematosus. The client now presents with sudden hematuria, variable proteinuria, and a decreased glomerular filtration rate. Which of the following is a probable diagnosis?

Acute nephritic syndrome

A 79-year-old woman reports a recent onset of "nearly constant heartburn." During the assessment interview, she states that she has "lots of aches and pains." She states that she is not on any prescription medications but often takes aspirin for pain. The nurse should suspect what diagnosis?

Acute gastritis

A male who has drunk excessive amounts of alcohol for 25 years is admitted to the hospital with cirrhotic liver disease. To assess the level of damage to the liver, which levels should be checked? Select all that apply.

AST GGT ALT

A nurse caring for a patient with an intestinal obstruction anticipates which of the following assessment findings? Select all that apply.

Abdominal pain Abdominal distension Vomiting

Irritable bowel syndrome is thought to be present in 10% to 15% of the population in the United States. What is its hallmark symptom?

Abdominal pain relieved by defecation with a change in consistency or frequency of stools

Which symptom is often observed in cases of peritonitis?

Abdominal rigidity

A client with multiple pain-related injuries to the back, knees, and hips is admitted with acute liver failure. Upon procuring a medication list, the nurse notes that the client is taking several over-the-counter medications that contain a preparation known to be the drug that most commonly causes liver failure. Which drug is in the client's medications?

Acetaminophen

A patient is being evaluated for kidney stones. The nurse anticipates the patient will manifest:

Acute onset of colicky or dull and achy intermittent flank pain

While on tour, a 32 year-old male musician has presented to the emergency department of a hospital after a concert complaining of severe and sudden abdominal pain. He admits to a history of copious alcohol use in recent years, and his vital signs include temperature 46.8°C (101.8°F), blood pressure 89/48 mmHg and heart rate 116 beats per minute. Blood work indicates that his serum levels of C-reactive protein, amylase and lipase are all elevated. Which of the following diagnoses would the care team suspect first?

Acute pancreatitis

A 34-year-old woman presents with an abrupt onset of shaking chills, moderate to high fever, and a constant ache in her lower back. She is also experiencing dysuria, urinary frequency, and a feeling of urgency. Her partner states that she has been very tired the last few days and that she looked like she may have the flu. What diagnosis is the most likely?

Acute pyelonephritis

A patient who developed acute pyelonephritis asks the nurse what caused the infection. Which should be included in thenurse's response? Select all that apply.

Acute pyelonephritis is caused by bacterial infection. • Escherichia coli is the causative agent in about 80% of cases. • Outflow obstruction, catheterization, and urinary instrumentation.

A warehouse worker is experiencing trouble with incontinence, especially when lifting heavy objects. What intervention is most appropriate for this client's needs?

Administration of a-adrenergic agonistic drugs as ordered

The oncology nurse is teaching a group of nursing students about risk factors for pancreatic cancer. Which of these does the nurse include in the teaching session? Select all that apply.

Age over 50 years Chronic pancreatitis Cigarette smoking

A patient is concerned that medication may damage his kidneys. Which factor(s) place the patient at most risk for developing a drug-related nephropathy? Select all that apply.

Age/elderly Prescription methicillin and other synthetic antibiotics Diuretics Procedures involving radiocontrast media Nonsteroidal anti-inflammatory drugs (NSAIDs)

A nurse is caring for a patient with diabetic glomerulosclerosis. The analysis is reviewed for the presence of which of the following manifestations?

Albumin

A patient with pancreatic cancer is admitted for portal hypertension in which they are symptomatic with ascites. Following paracentesis and removal of 7.5 L of ascitic fluid the nurse should anticipate that the physician will order which of the following medications to assist in maintaining an effective circulating fluid volume?

Albumin (Human) 5%.

Which of these substances should the nurse teach the client with pancreatitis is essential to avoid?

Alcohol

A 43 year-old male client has presented to the emergency department with vomiting that he claims is of a sudden onset. The client also states that the emesis has often contained frank blood in the hours prior to admission. His vital signs are stable with temperature 98.30 F, pulse 88, BP 140/87, and respiratory rate 18. Which of the following potential contributing factors would the health care team suspect first?

Alcohol consumption

The nurse is teaching a group of nursing students about alcohol-induced liver disease and the cytochrome P450 system (CYP P450). Which of these is correct for the nurse to include in the lesson?

Alcohol consumption enhances susceptibility to effects of drugs and toxins.

The radiologic studies of a client identify partial obstruction of the sphincter of the pancreas. The nurse recognizes that this is a result of:

Alcoholic beverages

A client with history of alcohol abuse is brought to the emergency department after a weekend of heavy drinking, experiencing right upper quadrant pain, anorexia, nausea, jaundice and ascites. The nurse identifies these as manifestations of what disorder?

Alcoholic hepatitis

A four year-old boy who has been deaf since birth and has bilateral cataracts has been brought to the emergency department by his mother because she noticed blood in the toilet after he last voided. Urinalysis confirms heavy microscopic hematuria as well as proteinuria. What will the healthcare team's initial differential diagnosis most likely be?

Alport syndrome

The nurse is caring for a client with hepatic encephalopathy who is receiving neomycin. Which of these does the nurse monitor to determine if a positive outcome to the medication has resulted?

Ammonia level

When assessing the client with acute pancreatitis, which of these diagnostic tests, consistent with the disease, does the nurse anticipate will be altered?

Amylase and lipase

The mother of a 19 week-old infant has brought her baby in for assessment to a pediatrician because of the baby's persistent weight loss and diarrhea. An intestinal biopsy has confirmed a diagnosis of celiac disease, and the child's mother is anxious to know what caused the disease. Which of the following aspects of the etiology of celiac disease would underlie the explanation that the physician provides?

An inappropriate T-cell mediated response results in increased levels of antibodies and an inflammatory response.

Which of the following accurately describes the etiology of stress incontinence?

An increase in intra-abdominal pressure which results in involuntary urination

Which additional physical finding would you anticipate seeing in a child suspected of having a Wilms tumor?

Anomalies of the genitourinary system

Sexual dysfunction in people with CKD is thought to be multifactorial. What are thought to be causes of sexual dysfunction in people with CKD? (Select all that apply.)

Antihypertensive drugs • Psychological factors • Uremic toxins

A nurse is reviewing the admission assessment data of a client diagnosed with acute gastritis. The nurse determines that the condition most likely occurred as a result of:

Arthritis treated with high levels of nonsteroidal anti-inflammatory (NSAIDs) agents

A patient with diabetes mellitus type 1 has bladder atony with dysfunction. Which of the following assessments indicates a primary concern for this patient?

Ascending urinary tract infection

An elderly patient presents with loose mucousy stools. The nurse suspects the patient has Clostridium difficile. What is a priority assessment for the nurse?

Ask the patient about his or her antibiotic use.

A patient is being screened for colorectal cancer with the fecal occult test. The nurse instructs the patient to avoid ingesting which of the following 3-7 days prior to the test? Select all that apply.

Aspirin Citrus fruits Red meats

A patient presents with a perforated peptic ulcer. Which of the following complications would be a priority for the nurse?

Assessment of a rigid, boardlike abdomen

A patient has bladder distension. Which of the following is the nurse's best action?

Assisting the patient to urinate

Most primary glomerular disease is caused by which of the following?

Autoimmunity

The form of polycystic kidney disease (PKD) that first manifests in the early infant period is most commonly characterized as which of the following?

Autosomal recessive

The nurse suspects that a newborn infant who presents with bilateral flank masses, impaired lung development, and oliguria may be suffering from which of the following disorders?

Autosomal recessive polycystic kidney disease (ARPD)

A 40-year-old man tells the nurse that his father died of gastric cancer and that he wants to do everything he can do to avoid the disease. Which recommendation should the physician provide?

Avoid smoked and preserved foods

A nurse is assessing a patient with suspected urine retention. Which of the following assessments help to confirm this diagnosis? Select all that apply.

Bladder distension Hesitancy Straining when initiating urination Frequency

A middle-aged man with diabetes reports that he must strain to urinate and that his urine stream is weak and dribbling. He also reports feeling that his bladder never really empties. The nurse knows that all of his complaints is likely caused by which of the following medical diagnosis?

Bladder atony with dysfunction

The nursing instructor, while teaching about renal function and disorders, informs the students that the most frequent form of urinary tract cancer is which of the following?

Bladder

Acute pyelonephritis is a result of which of the following?

Bacterial infection

Symptoms of gastric cancer include vague epigastric pain, which makes early detection difficult. The nurse would expect a patient to undergo which of the following diagnostic examinations to determine the location of the gastric cancer?

Barium x-ray

The nurse is caring for a patient who has had acute blood loss from ruptured esophageal varices. Which of the following does the nurse recognize is an early sign of prerenal failure?

Baseline urine output of 50 mL/hr that is now 10 mL/hr

A client has just been diagnosed with urinary incontinence. His primary nurse knows that the treatment/management of incontinence can include which of the following? Select all that apply.

Behavioral measures Surgical correction Pharmacologic measures Non-catheter devices to obstruct urine flow Indwelling catheter

Which clinical manifestations would you expect to see in an infant diagnosed with autosomal recessive polycystic kidney disease (ARPKD)?

Bilateral flank masses and impaired lung development

Treatment for Helicobacter pylori (H. pylori) requires combination therapy that includes which of the following medications? Select all that apply.

Bismuth salts Omeprazole Amoxicillin

The nursing instructor informs the students during a lecture on the lower urinary tract that which of the following controls the elimination of urine from the body?

Bladder

A nurse is caring for a patient with systemic lupus erythematosis (SLE). The patient asks why a urinalysis is necessary. The best answer is that a urinalysis would determine whether which of the following factors was present in the urine?

Blood or protein

When assessing a male client for manifestations of endocrine disorders related to hepatic failure, the nurse observes the client for which of these?

Breast enlargement

An adult client presents to the emergency department with manifestations of acute cystitis. For which would the nurse assess the client? Select all that apply.

Burning on urination Back discomfort Cloudy urine

What is the most common gram-negative bladder infection found in hospitalized clients?

Catheter-induced infection

A nurse correctly identifies which of the following to be approved treatments for a client who is admitted with a diagnosis of neurogenic bladder? Select all that apply.

Catheterization Bladder training Meds to manipulate bladder function Surgery

When caring for the client with pancreatic cancer, for which of these signs and symptoms does the nurse assess?

Central abdominal pain, jaundice and weight loss

A nurse has just completed the assessment of a client diagnosed with inflammatory diarrhea who is having multiple small stools that are bloody and has a fever. The client asks the nurse what is causing the diarrhea. The best response by the nurse would be:

C. difficile

A patient who has had an intestinal bypass has developed a kidney stone. Which of the following types of kidney stone does the nurse recognize that this patient will most likely be treated for?

Calcium

The nurse is caring for a patient with chronic renal failure who is on hemodialysis three times a week. In order to treat hyperphosphatemia and hypocalcemia, which of the following medications will the nurse administer to decrease absorption of phosphate from the gastrointestinal tract?

Calcium carbonate

A client presents with epigastric pain, a mild fever, nausea, and vomiting. His history shows a previous episode with similar symptoms that reverted in 24 hours. This time the pain is not subsiding. What diagnosis is the most likely?

Calculous cholecystitis

Select the body systems most commonly affected by chronic kidney disease in the elderly. Select all that apply.

Cardiovascular, cerebrovascular systems, skeletal

A school nurse is teaching a group of fourth-grade girls about personal hygiene. Important teaching points aimed at reducing the incidence of urinary tract infection (UTI) include which of the following? Select all that apply.

Careful hand washing • Wiping from front to back after a bowel movement • Avoiding bubble baths

Hospitalized patients are at the greatest risk of developing septicemia related to which procedure?

Catheter-associated bacteriuria

All diseases have risk factors. What is the most significant environmental risk factor for pancreatic cancer?

Cigarette smoking

A patient has just been admitted to a nursing unit with the diagnosis of obstructive jaundice. Which of the following assessment findings would the nurse expect to see in this patient? Select all that apply.

Clay-colored stools. Dark urine. Elevated conjugated bilirubin levels. Severe itching.

When assessing the nutritional intake of a client with generalized pancreatic cancer, which of these statements by the client is consistent with the disease and food intake?

Client states pain becomes worse with food intake.

A hospital client with a diagnosis of chronic renal failure has orders for measurement of her serum electrolyte levels three times per week. Which of the following statements best captures the relationship between renal failure and sodium regulation?

Clients with advanced renal failure are prone to hyponatremia because of impaired tubular reabsorption.

Crohn's disease is recognized by sharply demarcated, granulomatous lesions that are surrounded by normal-appearing mucosal tissue. The nurse recognizes these lesions to be defined by which of the following descriptions?

Cobblestone

A client is admitted to a nursing unit with severe edema. The nursing student caring for this client overhears the physician and a medical student talking about the client's albumin level. When discussing the flow of fluids into and out of cells, albumin plays a significant role in which pressure gradient?

Colloidal osmotic

An elderly patient has been placed on a broad-spectrum antibiotic for a recurrent urinary tract infection. Which of the following potential problems would the nurse anticipate in this patient?

Colonization of Clostridium difficile

A patient diagnosed with ulcerative colitis has been experiencing more than six bloody stools daily with evidence of toxicity. The nurse should question which of the following orders from the physician?

Colonoscopy

Incontinence can be transient. What are the causes of transient urinary incontinence? (Select all that apply.)

Confusional states Stool impaction Recurrent urinary tract infections

Chronic kidney disease (CKD) can provoke many cardiovascular changes. The nurse will monitor for which of the following in a client with CKD? Select all that apply.

Congestive heart failure • Hypertension • Impaired platelet function • Pericarditis

A male infant who is 48 hours postpartum is undergoing phototherapy for the treatment of jaundice and accompanying high levels of bilirubin. Place the following components of the production of bilirubin in the correct chronological order. Use all the options.

Conjugated bilirubin Urobilinogen Red blood cells Biliverdin Free bilirubin

Which statements identify bladder function? Select all that apply.

Control of function involves the autonomic and somatic nervous systems. The sympathetic nervous system promotes bladder filling. The parasympathetic nervous system promotes emptying.

A 24-year-old woman undergoing a premarital screening test is found to have elevated levels of AST, ALT, and IgG, but no antibody-specific markers for viral hepatitis. A liver biopsy reveals inflammation and cellular damage. Which of the following treatments is most likely to be effective for her?

Corticosteroids and immunosuppressant drugs

A 24-year-old woman undergoing a screening test is found to have elevated levels of AST, ALT, and IgG, but no antibody-specific markers for viral hepatitis. A liver biopsy reveals inflammation and cellular damage. Which treatment is most likely to be effective for her?

Corticosteroids and immunosuppressant drugs

A 28-year-old man presents with complaints of diarrhea, fecal urgency, and weight loss. His stool is light-colored and malodorous, and it tends to float and be difficult to flush. He has also noted tender, red bumps on his shins and complains of pain and stiffness in his elbows and knees. Sigmoidoscopy reveals discontinuous, granulomatous lesions; no blood is detected in his stool. Which of the following diagnoses would his care team first suspect?

Crohn disease

A young man presents reporting diarrhea, fecal urgency, and weight loss. His stool is light-coloured and malodorous, and it tends to float and be difficult to flush. He has also noted tender, red bumps on his shins and reports pain and stiffness in his elbows and knees. Sigmoidoscopy reveals discontinuous, granulomatous lesions; no blood is detected in his stool. Which diagnosis would his care team first suspect?

Crohn disease

The nurse is caring for a patient with chronic diarrhea. She knows that this condition could be caused by which of the following? Select all that apply.

Crohn disease, Ulcerative colitis, Lactase deficiency, and Fecal impaction.

A nurse reading a sigmoidoscopy report notes that a patient was found to have skip lesions. The nurse interprets this as an indication of which of the following?

Crohn's disease

The nurse is conducting pre-operative teaching for a patient with bladder cancer who is scheduled to undergo surgical creation of an alternative bladder reservoir. The nurse determines that the patient is understanding the pre-operative teaching when the patient identifies the surgical treatment as which of the following?

Cystectomy

A client tells the doctor that every time he eats something made with wheat, barley, and/or rye, he gets sick. The doctors suspects the client has celiac sprue. The physician should further assess the client for: Select all that apply.

Diarrhea Abdominal pain Iron deficiency

A new client presents with elevated BUN, systemic edema, a BP of 145/93 mm Hg, recurrent infections, and a GFR of 51 mL/min/1.73 m2. What treatment should the nurse anticipate?

Dialysis

A nursing instructor who is teaching students about urinary incontinence in older adults suggests that an easy and best way to remember the transient and treatable causes of urinary incontinence is to use which of the following acronyms?

DIAPPERS

Which physiologic change in the elderly population contributes to urinary incontinence?

Decline in detrusor muscle function

The nurse is caring for a client with liver disease who has edema throughout the body. When reviewing the medical record, the nurse recognizes which of these altered diagnostic tests is consistent with development of edema?

Decreased albumin

The nurse is preparing to assess a client who has just been admitted to the hospital with a diagnosis of prerenal failure. Which would the nurse expect the client to manifest? Select all that apply.

Decreased urinary output Increased BUN BUN-to-serum creatinine ratio of greater than 20:1

A client with persistent, recurring episodes of epigastric and upper left quadrant pain and anorexia, nausea, vomiting, constipation, and flatulence has been diagnosed with chronic pancreatitis. What is the cause of the symptoms of chronic pancreatitis?

Destruction of both the endocrine and exocrine pancreas

While studying about the process of urination, the nursing student learns that which of the following is known as the "muscle of micturition"?

Detrusor muscle

When explaining about the passage of urine to a group of nursing students, the clinic nurse asks them which muscle is primarily responsible for micturition? Their correct reply is the:

Detrusor.

A client is diagnosed with chronic kidney disease (CKD). The nurse will monitor this client for which gastrointestinal signs and symptoms? Select all that apply.

Early morning nausea Gastrointestinal ulceration Metallic taste Anorexia

A client is being treated for chronic kidney disease (CKD. . One of the nurse's responsibilities is to explain to the client the need to keep her blood pressure under control. Why is blood pressure control so important in CKD clients?

Elevated blood pressure will exacerbate nephron loss and accelerate renal failure.

Following the diagnosis of acute renal failure, the nurse knows that one of the earliest manifestations of residual tubular damage is which of the following lab/diagnostic results?

Elevated blood urea nitrogen (BUN).

A 53-year-old woman with a history of chronic alcohol abuse but without visible jaundice comes to the clinic complaining of nausea and weakness. She admits to taking acetaminophen for persistent headaches but denies exceeding the recommended daily dose; she has not taken any other medications. She is suspected of having acetaminophen toxicity. Which of the following diagnostic test findings would implicate a different cause of her symptoms?

Elevated serum HBsAG level

A client with a history of chronic alcohol use but without visible jaundice comes to the clinic reporting nausea and weakness. She admits to taking acetaminophen for persistent headaches but denies exceeding the recommended daily dose; she has not taken any other medications. She is suspected of having acetaminophen toxicity. Which diagnostic test finding would suggest a different cause of her symptoms?

Elevated serum HBsAG level

Which laboratory findings should you expect to see in a patient diagnosed with nephritic syndrome?

Elevated urine protein and hypoalbuminemia

A patient is admitted with chronic gastritis. The nurse expects which of the following invasive tests to be performed to establish the presence of Helicobacter pylori (H. pylori)?

Endoscopic biopsy

An older male comes to the clinic with the chief complaint of having difficulty voiding. The physician diagnoses him with a lower urinary tract obstruction and stasis. Which of the following should the nurse suspect to be the most frequent cause of this client's problem?

Enlargement of prostate gland

An newborn has been diagnosed with renal agenesis. The nurse caring for the mother mentions some of the classic signs/symptoms of this disease. Which characteristic should the nurse mention to the parents as it relates to this disease? Select all that apply.

Epicanthic folds (skin of the upper eyelid) are present Low-set ears Receding chin

Which of the following is the most common cause of lower uncomplicated urinary tract infections?

Escherichia coli

The physician has just completed an assessment on a client diagnosed with cirrhosis. The client asks if she is at risk for any serious complications. Which would be the best response?

Esophageal varices

Parents of a 20-month-old infant report that he refuses food or eats poorly and that he grimaces when he swallows. He also is irritable and cries a lot. The mother is worried that he ate something inappropriate this morning, because he vomited something that looked like coffee grounds. Which of the following health problems would the care team first suspect?

Esophagitis from gastrointestinal reflux

Urinary incontinence can be a problem with the elderly. One method of treatment is habit training, or bladder training. When using this treatment with an elderly person, how frequently should he or she be voiding?

Every 2 to 4 hours

A nurse is teaching a client diagnosed with Crohn disease about potential complications. The most appropriate information for the nurse to include would be:

Fistula formation

Crohn's disease has a distinguishing pattern in the gastrointestinal (GI) tract. The surface has granulomatous lesions surrounded by normal-appearing mucosal tissue. A complication of the pattern includes which of the following?

Fistula formation

When comparing the symptomology of Crohn disease with that of ulcerative colitis, which symptoms are generally characteristic of only Crohn disease? Select all that apply.

Fistulas. Perianal ulcers.

The nurse is caring for a patient with a sacral cord injury that resulted in loss of the perception of bladder fullness and loss of voluntary urination. When planning care for the patient, the nurse should include nursing interventions for which of the following?

Flaccid bladder dysfunction

A nurse on an acute medical unit of a hospital has admitted a 62 year-old female from the emergency department who has been diagnosed with acute pyelonephritis. Which of the following statements most accurately conveys an aspect of the knowledge base that the nurse needs to perform adequate care and teaching?

Flank pain, dysuria and nausea and vomiting are likely assessment findings.

The nurse is assessing a client with acute pancreatitis who denies use of alcohol. When teaching the client about additional causes of acute pancreatitis, the nurse includes which of these in the discussion? Select all that apply.

Gallstones Increased cholesterol Abdominal trauma

The health care provider is reviewing lab results of a client. Select the test that is the best measurement of overall kidney function?

Glomerular filtration rate (GFR)

A patient has passed a kidney stone composed of uric acid (urate). Which of the following pathological conditions is a contributing factor for the development of this type of kidney stone?

Gout

A client informs the nurse that she is afraid of developing bladder cancer because her mother had it. She asks the nurse what signs and symptoms are present with this cancer. What does the nurse tell the client is the most common sign of bladder cancer?

Gross hematuria

Acute postinfectious glomerulonephritis, as its name implies, follows an acute infection somewhere else in the body. What is the most common cause of acute postinfectious glomerulonephritis?

Group A Β-hemolytic streptococci

In the balance of secretions in the gastric mucosa by the parietal cells, which of the following ions is produced to buffer the production of hydrochloric acid?

HCO3-

Older adults often have other chronic diseases that influence the early symptoms and signs of renal dysfunction. The nurse knows that which of the following can be the dominant clinical events in older adults with early kidney disease? Select all that apply.

Heart failure Hypertension

A nurse is assessing a client for early manifestations of chronic kidney disease (CKD). Which would the nurse expect the client to display?

Hypertension

Which of the following have the potential to cause chronic kidney disease? (Select all that apply.)

Hypertension • Diabetes • Glomerulonephritis

A client tells the nurse that he is concerned about developing hepatitis after being exposed to contaminated feces, saliva, and food. The nurse is aware that the client is at risk for:

Hepatitis A

The nurse is caring for a client with hepatitis A who is tired of feeling ill. Which of these does the nurse convey to the client?

Hepatitis A symptoms last for 8 weeks.

A baby is born to a mother with active hepatitis B. Which of the following medications does the nurse anticipate administering today?

Hepatitis B immune globulin (HBIG).

A 29 year-old female has been admitted to the emergency department following a suicide attempt by overdose of acetaminophen. What physiologic changes in the client's liver causing which of these altered diagnostic results does the nurse anticipate?

Hepatocellular necrosis evidenced by increased ALT and AST levels.

Which factor contributes to the development of polycystic kidney disease?

Hereditary mutations in polycystin I and II

A neonate is undergoing phototherapy for the treatment of jaundice and accompanying high levels of bilirubin. The infant has developed jaundice because of what factors? Select all that apply.

High bilirubin production Limited ability to excrete bilirubin

When assessing a patient with acute pylenephritis, the nurse would expect the patient to exhibit which of the following symptoms?

High fever Flank tenderness Nausea and vomiting Chills

The physician suspects a client may have developed pancreatitis, and the physician has ordered laboratory blood work. Diagnosis-confirming results would identify:

High serum amylase and lipase

A patient is diagnosed with Crohn's disease. The nurse instructs the patient on which of the following types of dietary needs?

High-calorie, vitamin, and protein diet

A nurse is caring for a patient who has a recent history of passing calcium urinary stones. Which of the following is a priority nursing consideration for this patient?

Hydration

A nurse is administering medications to a patient with a kidney stone. Which medication does the nurse administer that will lower urinary calcium by increasing tubular reabsorption to decrease the amount of calcium in the urine?

Hydrochlorothiazide (HCTZ)

A client has a dilated renal pelvis due to obstruction of urine outflow from the kidney. The pressure of filtrate formation is damaging the renal structures. Which condition is most likely?

Hydronephrosis

When caring for the client with acute pancreatitis, which of these alterations does the nurse recognize is consistent with the disease?

Hyperglycemia

The nurse is caring for the client with pancreatic cancer. The nurse monitors the client for which of these complications?

Hyperglycemia due to inability to synthesize insulin

A child has received a kidney transplant at the age of 3. Knowing he will be on immunosuppressive agents like corticosteroids, the nurse should educate the parents about which long-term side effects? Select all that apply.

Hypertension • Growth retardation • Development of cataracts

A client has acute tubular necrosis (ATN) caused by myoglobinuria. The nurse knows that myoglobinuria can result from which of the following? Select all that apply.

Hyperthermia • Muscle damage • Alcohol abuse • Prolonged seizures

A 68 year-old woman with a new onset of vascular dementia has recently begun retaining urine. Which of the following physiological phenomena would her care providers most realistically expect to be currently occurring as a result of her urinary retention?

Hypertrophy of the bladder muscle and increased bladder wall thickness.

Signs of nephrotic syndrome include proteinuria, edema, hyperlipidemia, and which of the following?

Hypoalbuminemia

A nine year-old boy has been diagnosed with the nephritic syndrome. Place the following stages in the development of his health problem in ascending order. Use all the options.

Hypoalbuminemia Increased glomerular membrane permeability. Decreased colloidal osmotic pressure Proteins escape from plasma to glomerular filtrate. Accumulation of fluid in interstitial tissue (edema).

The nurse is caring for the client with hepatocellular carcinoma. Which of these does the nurse recognize is a cause of this disease?

Illness with hepatitis B or C

Which stresses can cause injury to the kidney glomerulus? Select all that apply.

Immunologic Nonimmunologic Heredity Diabetes

The nurse is teaching a client about the different treatments used with bladder cancer. Which type of therapy does she inform the client can be done by instilling directly into the bladder?

Immunotherapuetic agents

A client diagnosed with CKD has begun to experience periods of epistaxis and developed bruising of skin and subcutaneous tissues. The nurse recognizes these manifestations as:

Impaired platelet function

The health care provider has completed the assessment of a client who presented to the emergency department with jaundice. The provider determines that the jaundice is a result of:

Impaired uptake of bilirubin by the liver

When caring for the client with Laennec's cirrhosis, the nurse recognizes which of these is an expected etiology of jaundice?

Impaired uptake of billirubin

Skeletal disorders frequently accompany chronic kidney disease (CKD). The nurse recognizes which of the following statements regarding bone turnover in CKD are correct? Select all that apply.

In high-bone-turnover, there is both increased bone formation and increased resorption. High-bone-turnover produces porous, coarse-fibered bone. Metabolic acidosis of CKD can affect bone turnover. Both high and low turnover present risk for fracture.

A patient sustained acute tubular injury approximately 2 hours ago. Which of the following phases of this disorder does the nurse recognize that the patient is in at this time?

Initiating phase

The nurse recognizes that which of these individuals are at risk for hepatitis B? Select all that apply.

Injection drug users Those having unprotected sex

In hemodialysis, access to the vascular system is most commonly through what?

Internal arteriovenous fistula

A client is concerned about acquiring hepatitis A and asks the nurse who should receive the HAV vaccine. The best response would be: Select all that apply.

International travelers visiting to regions where sanitation is poor Homosexually active men Users of illicit drugs Workers employed in food handling

The client in the GI clinic tells the nurse she is concerned she has something wrong with her gallbladder like what her friend with gallstones and inflammation had. Which of these complaints does the nurse recognize that supports the client's concern?

Intolerance to greasy food; burping

A client has acute pyelonephritis. The nurse will monitor the client for development of which of the following?

Intrarenal failure

A 45-year-old man is being treated for bladder cancer. Which therapy would have the least complications?

Intravesical chemotherapy

When educating the patient about possible treatments following surgery for bladder cancer, the nurse might include which of the following chemotherapy options? Select all that apply.

Intravesical chemotherapy with doxorubicin (Adriamycin). Bacillus Calmette-Guerin (BCG) vaccine. Endocan, a tumor angiogenesis inhibitor.

Which of the following can be said about acute drug-induced interstitial nephritis? Select all that apply.

It is an autoimmune disease. It may be related to a sensitivity to sulfonamide drugs. It manifests as a fever, hematuria, mild proteinuria, and possibly a rash. It may result in acute renal failure.

A 67-year-old retired textile worker has developed bladder cancer. His history shows that he smoked for 25 years and is 14 kg overweight. What conclusion can the nurse best draw about the etiology and pathophysiology of his bladder cancer?

It is caused by carcinogens that are excreted in the urine and stored in the bladder.

A nurse is giving discharge instructions for a patient who was diagnosed with acute pyelonephritis 3 days previously. Which of the following is important for the nurse to discuss when instructing the patient?

It is important that the patient take the prescribed antibiotic for the duration of the prescription.

Which statements are true regarding a Wilms tumor? Select all that apply.

It is the most common childhood malignant tumor. It presents with an abdominal mass Treatment includes chemotherapy, radiation and surgery

A client is prescribed erythromycin for an infection. What manifestations will the nurse recognize that indicate the onset of drug-induced cholestasis?

Jaundice and pruritus

A female client asks the nurse if there is any noninvasive treatment to help with the involuntary loss of urine that occurs when she coughs or sneezes. Which is the best response by the nurse?

Kegel exercises

Vitamin D metabolism is deranged in clients with chronic kidney disease (CKD). The nurse recognizes that which of the following statements regarding vitamin D is correct?

Kidneys convert inactive vitamin D to its active form, calcitriol.

The liver has many jobs. One of the most important functions of the liver is to cleanse the portal blood of old and defective blood cells, bacteria in the bloodstream, and any foreign material. Which cells in the liver are capable of removing bacteria and foreign material from the portal blood?

Kupffer cells

The nurse is assessing a client exposed to viral hepatitis who is thought to be in the prodromal phase. When assessing the client, which of these symptoms does the nurse recognize are consistent with this phase? Select all that apply.

Lack of appetite Lack of energy Myalgia

When educating a client who has been experiencing some manifestations of liver disease about the lab/diagnostic procedures available, which procedure provides the most information about abdominal masses and possible staging of a liver cancer?

Laparoscopic liver biopsy

Wilms tumor is a tumor of childhood. It is usually an encapsulated mass occurring in any part of the kidney. What are the common presenting signs of a Wilms tumor?

Large asymptomatic abdominal mass and hypertension

When caring for the client with portal hypertension and ascites, which of these dietary interventions does the nurse suggest to prevent the progression of fluid accumulation?

Limit intake of sodium.

What laboratory markers are most commonly used to diagnose acute pancreatitis?

Lipase and amylase

A nurse is reviewing laboratory results of a client that were ordered to assess liver function .The results indicating injury to liver cells would include:

Liver enzymes

When caring for a group of clients with alcoholism and cirrhosis, which of these does the nurse recognize are potential complications of the disease? Select all that apply.

Liver failure • Bleeding esophageal varices • Kidney failure

The nurse assessing a renal failure patient for encephalopathy caused by high uremic levels may observe which of the following clinical manifestations?

Loss of recent memory and inattention.

The nurse should assess which of the following when assessing for the initial presence of edema in a patient with nephrotic syndrome?

Lower extremities

Diverticulitis is the herniation of tissue of the large intestine through the muscularis layer of the colon. It is often asymptomatic and is found in approximately 80% of people over the age of 85. Diverticulitis is often asymptomatic, but when symptoms do occur, what is the most common complaint of the client?

Lower left quadrant pain with nausea and vomiting

A client is diagnosed with renal failure. The nurse must monitor for failure of which of the following functions? Select all that apply.

Maintenance of body water Electrolyte balance Acid-base balance Removal of metabolic waste from blood

A stroke patient is having difficulty swallowing food and beverages. The patient complains that it feels like "the food is sticking to the back of their throat." Given this information, the priority nursing interventions would be to:

Make the patient "nothing per os" (NPO) and call the physician.

Crohn's disease not only affects adults but also can occur in children. The nurse assesses for which of the following major manifestations in children with Crohn's disease?

Malnutrition

The nurse is caring for a client with asterixis. Which of these additional assessments should the nurse make to help a diagnosis of hepatic encephalopathy?

Mental status

A nurse is explaining the clinical manifestations of diabetic nephropathy (diabetic glomerulosclerosis) to a patient. Which would be the most important information for the nurse to provide?

Microalbuminemia is a predictor of future nephropathies.

Which layer is produced by the bladder epithelia that binds water and forms a protective barrier between the bladder wall and the components of urine, thus protecting the bladder from carcinogens and infective organisms?

Mucin

A 20 year-old woman has visited her family physician due to occasional bouts of bloody diarrhea over the past several weeks, a phenomenon that she experienced two years prior as well. Her physician has diagnosed her with ulcerative colitis based on her history and visualization of the affected region by colonoscopy and sigmoidoscopy. Which of the following pathophysiological phenomena is most likely to underlie the client's health problem?

Mucosal hemorrhages that have developed into crypt abscesses, which have in turn necrotized and ulcerated.

A 35-year-old female ultramarathon runner is admitted to hospital following a day-long, 80-km race because her urinary volume is drastically decreased and her urine is dark red. Tests indicate that she is in the initiating phase of acute tubular necrosis. Why is her urine red?

Myoglobinuria

A 35-year-old female ultramarathon runner is admitted to the hospital following a day-long, 50-mile race because her urinary volume is drastically decreased and her urine is dark red. Tests indicate that she is in the initiating phase of acute tubular necrosis. Why is her urine red?

Myoglobinuria

A patient is admitted with an intestinal obstruction with abdominal bloating. Which of the following types of treatment does the nurse know will help decompress the bowel?

Nasogastric tube to suction

A nurse is caring for a patient who is diagnosed with acute poststreptococcal glomerulonephritis. Which of the following is the most common clinical presentation of this condition?

Nephritic syndrome

The health care provider has prescribed an aminoglycoside (gentamicin) for a client. The nurse is aware that the client is at risk for:

Nephrotoxic acute tubular necrosis

An elderly woman comes to the hospital and is diagnosed with urinary obstruction and retention. Which of the following symptoms would the nurse expect this client to demonstrate? Select all that apply.

Nladder distention • Hesitancy • Frequency • Overflow incontinence

A 45-year-old client with chronic kidney disease (CKD) voices concern about her dialysis treatment. The client would like to work and spend time with her family. Which type of dialysis will best fit this client's lifestyle?

Nocturnal intermittent peritoneal dialysis (NIPD)

Diarrhea is described as a change in frequency of stool passage to a point where it is excessively frequent. Diarrhea can be acute or chronic, inflammatory, or noninflammatory. What are the symptoms of noninflammatory diarrhea? (Select all that apply.)

Nonbloody stools, periumbilical cramps, and nausea and/or vomiting

A patient with a history of peptic ulcer disease presents to the emergency department with the following symptoms: early satiety, feeling of epigastric fullness and heaviness after meals, weight loss, and vomiting. The nurse suspects that the peptic ulcer has caused which of the following problems?

Obstruction

A client has been diagnosed with cholelithiasis. The nurse is aware the client is at risk for: Select all that apply.

Obstruction of bile flow • Biliary colic • Acute cholecystitis • Chronic cholecystitis

The nurse is teaching an adult client about home treatment for uncomplicated diarrhea. The most appropriate information for the nurse to provide would be:

Oral replacement therapy (ORT)

A 55 year-old man has made an appointment to see his family physician because he has been awakening three to four times nightly to void, and often has a sudden need to void with little warning during the day. What is the man's most likely diagnosis and possible underlying pathophysiological problem?

Overactive bladder that may result from both neurogenic and myogenic sources.

When assessing a client with acute cholecystitis, the nurse anticipates the client's report of pain will be consistent with which of these descriptions?

Pain in the right upper quadrant referred to the same shoulder

When teaching a community education class about the 7 warning signs of cancer, the nurse will note that the most common sign of bladder cancer is:

Painless bloody urine

A client asks the nurse what the most common sign/symptom of bladder cancer is. Which is the best response by the nurse?

Painless hematuria

The nurse is caring for a client with metastatic pancreatic cancer who is to undergo a surgical resection and radiation therapy. Which of these does the nurse recognize is the goal of these interventions?

Palliation of symptoms

A patient who has suffered a spinal cord injury at C4 is experiencing a sudden change in condition. Their BP is 186/101; heart rate 45; profusely sweating and complaining of "not feeling right." The nurse should:

Palpate their bladder for overdistention.

When teaching the client about the cause of acute pancreatitis, which of these points should be included in the teaching session?

Pancreatic enzymes are activated before leaving the pancreas causing injury.

A patient is concerned about the possibility of having bladder cancer after his brother was diagnosed with it 2 years ago. Which of the following assessment data obtained by the nurse would indicate that the patient should be screened for this disease?

Patient reports that he occasionally has blood in his urine but has no pain with it.

The nurse is evaluating her patient's risk for an urinary tract infection. Which of the following patients has the highest risk?

Patient with obstructed urinary outflow from the kidney

The incidence of stomach cancer has significantly decreased in the United States, yet it remains the leading cause of death worldwide. The nurse understands the reason for the high mortality rate in stomach cancer is because of which of the following reasons?

Patients have few early symptoms of the disease.

Upon admission, a patient tells the nurse that he takes aspirin every 4 hours every day. The nurse determines that this patient is at risk for which of the following?

Peptic ulcer

Peptic ulcers can affect one or all layers of the stomach or duodenum. Ulcers that penetrate through the muscularis layer are classified as which of the following types of ulcer?

Perforation

A client has been admitted to the hospital with an exacerbation of peptic ulcer disease. The nurse is aware the client is at risk for: Select all that apply.

Perforation • Hemorrhage • Obstruction

A nurse is caring for a patient in spinal shock. Which of the following interventions is appropriate in relation to the patient's urinary status?

Perform intermittent catheterization

An 87 year-old male resident of an assisted living facility has been consistently continent of urine until the last several weeks. Which of the following actions by the care providers at the facility is the most likely priority?

Performing a physical examination and history to determine the exact cause and character of the incontinence.

A patient with stage 5 chronic kidney disease (CKD) is presenting with fever and chest pain, especially when taking a deep breath. The nurse detects a pericardial friction rub on auscultation. Which of the following conditions does the nurse suspect is common with this stage of kidney disease?

Pericarditis

A client has developed a ruptured appendix. The nurse is aware the client is at high risk for:

Peritonitis

A patient is admitted with an abrupt onset of referred pain to the epigastric area, with an episode of nausea. On the nurse's initial assessment, the patient is lying still and taking shallow breaths, with a rigid abdomen. Which of the following problems is the patient experiencing?

Peritonitis

An older patient presents with a perforation of a peptic ulcer. The nurse will monitor for signs and symptoms of which of the following problems?

Peritonitis

A patient in renal failure has marked decrease in renal blood flow caused by hypovolemia, caused by gastrointestinal bleeding. The nurse is aware that this form of renal failure can be reversed if the bleeding is under control. Which of the following forms of acute renal injury does this patient have?

Prerenal failure

The nurse is caring for a client with viral hepatitis who has general malaise, easy fatigability, arthralgia, and anorexia. These manifestations correspond with what stage of the disease?

Prodromal period

When teaching a client at the GI clinic about chronic pancreatitis, which of these does the nurse relate is the long term result of this condition?

Progressive destruction of the organ

A 71-year-old client reports to the nurse that he often notices a pink tinge to his urine. Upon further questioning, he states that he experiences no pain when voiding and has not noticed any change in the frequency of his voiding. Which response by the nurse is best?

Promptly report this finding to the client's primary care provider.

The nurse is reviewing the diagnosis of four male clients. Select the diagnosis that places the clients at risk for developing postrenal kidney failure.

Prostatic hyperplasia

Which of the following substances would not be found in glomerular filtrate?

Protein

The nurse is reviewing the lab results of a patient with suspected nephrotic syndrome. The nurse anticipates that the results to include:

Protein in the urine

A patient has recently been diagnosed with H. pylori gastritis. The nurse knows that this form of gastritis is usually treated with a combination of an antibiotic and:

Proton pump inhibitors.

After several months of persistent heartburn, a 57 year-old female client has been diagnosed with gastroesophageal reflux disease (GERD). Which of the following treatment regimens is likely to best address the woman's health problem?

Proton pump inhibitors; avoiding large meals; remaining upright after meals

After several months of persistent heartburn, a 57-year-old female client has been diagnosed with gastroesophageal reflux disease (GERD). Which treatment regimen is likely to best address the woman's health problem?

Proton pump inhibitors; avoiding large meals; remaining upright after meals

Which symptom occurs in a client with chronic kidney disease (CKD) as a result of elevated serum phosphate levels and the development of phosphate crystals that occur with hyperparathyroidism?

Pruritus

Acute pyelonephritis is an infection of the renal parenchyma and renal pelvis. What is a common cause of acute pyelonephritis?

Pseudomonas species

The nurse recognizes that acute renal injury is characterized by which of the following?

Rapid decline in renal function

The nurse reviews the lab results for a patient who has advanced autosomal dominant polycystic kidney disease (ADPKD). The patient 's hemoglobin is 8.8 g/dL. The nurse suspects this lab value is related to which of the following causes?

Reduced production of erythropoietin

A 72-year-old patient is scheduled for a kidney transplant. The nurse knows that which aspect of advanced age has a positive effect on the success of kidney transplant survival?

Reduction in T-lymphocyte function

Which of the following phenomena contributes to the difficulties with absorption, distribution and elimination of drugs that is associated with kidney disease?

Reductions in plasma proteins increase the amount of free drug and decrease the amount of protein-bound drug.

A client who is admitted to the hospital receives a diagnosis of lower urinary tract obstruction. What does the nurse pick to be the immediate treatment for this problem?

Relief of bladder distention

Celiac disease commonly presents in infancy as failure to thrive. It is an inappropriate T-cell-mediated immune response, and there is no cure for it. What is the treatment of choice for celiac disease?

Removal of gluten from the diet

Which of the following is the most common cancer of the kidney?

Renal cell carcinoma

A client is diagnosed with chronic kidney disease (CKD). The nurse knows that which of the following statements regarding CKD are correct? Select all that apply

Renal damage is irreversible. Hypertension is a major cause.

A client with chronic hepatitis B is undergoing diagnostic testing for possible liver cancer. Upon taking a history and performing a physical exam, the nurse can correlate the possible cancer diagnosis with which clinical manifestations? Select all that apply.

Reports "abdominal fullness and bloating." States "My abdomen is getting larger since and I can no longer get my belt buckled."

A client diagnosed with chronic kidney disease (CKD) is experiencing nausea and vomiting. Which would be the best instruction for the nurse to provide?

Restrict intake of dietary protein

The nurse is performing physical assessment of the gastrointestinal tract. In which of these areas does the nurse place the hands for palpation of the liver?

Right upper quadrant

The nurse is assessing a client who has just been admitted to the unit with a diagnosis of cholelithiasis. The nurse is aware that the client may manifest:

Right upper quadrant pain

The GFR is considered to be the best measure of renal function. What is used to estimate the GFR?

Serum creatinine

The nurse assesses a patient for prerenal failure. Which conditions increase the risk for developing this type of kidney failure? Select all that apply.

Severe burns Septic shock Severe dehydration

The nurse working in the nursery is caring for a 4-day-old infant with autosomal recessive polycystic kidney disease (ARPKD). Which physical assessment findings would the nurse associate with this disease process? Select all that apply.

Severe hypertension 120/90 • Hypoventilation due to impaired lung development • Palpable masses located in the flank area bilaterally

Acute tubular necrosis (ATN) is the most common cause of intrinsic renal failure. One of the causes of ATN is ischemia. What are the most common causes of ischemic ATN? (Select all that apply.)

Severe hypovolemia, Burns, and overwhelming sepsis.

A client who has been diagnosed with acute symptomatic viral hepatitis is now in the icteric period. The nurse would expect the client to manifest:

Severe pruritus and liver tenderness

A 22 year-old female with a history of intermittent flank pain, repeated UTIs and hematuria has been diagnosed with autosomal dominant polycystic kidney disease (ADPKD). Which of the following phenomena has most likely contributed to the development of this diagnosis?

She has inherited a tendency for epithelial cell in her tubules to proliferate inappropriately.

Inflammatory bowel disease (IBD) is used to designate two related inflammatory intestinal disorders: Crohn's disease and ulcerative colitis. The nurse recognizes the difference between the distribution pattern between Crohn's disease and ulcerative colitis. Which of the following patterns describes Crohn's disease?

Skip lesions

The nurse is assessing a patient with diabetic nephropathy whose blood pressure is 124/80. The patient smokes two packs of cigarettes a day and consumes a diet high in saturated fats and sodium. The nurse's plan of care should focus on which of the following to decrease the patient's risk for progression of the diabetic nephropathy?

Smoking cessation program

A patient is admitted with a peptic ulcer. The nurse's priority on admission is to assess for which of the following? Select all that apply.

Smoking history • Family history • Alcohol usage

One form of renal tubular acidosis (RTA) results from aldosterone deficiency or resistance to its action, which leads to impaired reabsorption of which of the following electrolytes?

Sodium

Drug-related nephropathies occur all too often. They involve functional and/or structural changes to the kidney after exposure to a drug. What does the tolerance to drugs depend on?

State of hydration

The cause of gastric carcinomas has been influenced by which of the following factors? Select all that apply.

Strain of Helicobacter pylori (H. pylori) • Environmental factors • Family history

A nurse is caring for a patient who has been diagnosed with kidney colic and has not yet passed the stone. Which of the following interventions should the nurse be sure to include when planning the care for this patient?

Strain the urine.

Which of the following are appropriate interventions in the care of a patient diagnosed with renal calculi? Select all that apply.

Straining the client's urine • Addressing the client's pain • Keeping track of intake and output

A client who has a problem with incontinence loses a small amount of urine every time she coughs or sneezes. This type of incontinence is known as which of the following?

Stress

The patient who has experienced third-degree burns is susceptible to which of the following specific types of gastrointestinal (GI) ulceration?

Stress

A 40-year-old mother of three reports incontinence. Her physician suggests Kegel exercises, because they strengthen the pelvic floor muscles. Kegel exercises are most likely to help which type of incontinence?

Stress incontinence

A patient is describing difficulty with urinating and informs the nurse that every time she coughs or laughs, she urinates and has begun to wear a thin pad. Which of the following types of urinary incontinence is the patient describing?

Stress incontinence

Because they strengthen the pelvic floor muscles, Kegel exercises are most likely to help:

Stress incontinence

A nurse administering a patient's medication tells the patient that a proton pump inhibitor has been added. When the patient asks the purpose of the medication, the nurse responds that it is to prevent which of the following?

Stress ulcer

A client is admitted to the hospital with a suspected diagnosis of strangulated bowel. The nurse anticipates the client will need:

Surgery to release the bowel

The nurse working on a GU floor caring for clients who have bladder cancer identifies which of the following to be acceptable treatments for this cancer? Select all that apply.

Surgical removal• Radiation therapy• Chemotherapy

A client is undergoing diagnostic testing for possible liver failure. While educating a group of nursing students, the nurse asks them what the clinical manifestations of liver failure include. Which answers are accurate? Select all that apply.

Sweet, musty breath. • Multiple bruising noted on body

A client presents to the emergency department with suspected pancreatitis. Select the manifestations the nurse would assess the client for: Select all that apply.

Tachycardia Fever Tenderness Epigastric pain

A client with chronic kidney disease (CKD) is anemic. The nurse will attempt to alleviate the anemia in order to prevent which of the following? Select all that apply.

Tachycardia • Fatigue • Decreased myocardial oxygen

Parents have brought their seven year old child into the emergency room with abdominal pain. Which of the following clinical manifestations would lead the healthcare team to suspect the child has appendicitis? Select all that apply.

Tenderness in right lower quadrant with palpation.• Rebound tenderness in inguinal areas with palpation.

A client has experienced severe hemorrhage and is in prerenal failure. The nurse anticipates the laboratory results of the client's BUN and serum creatinine to identify:

The BUN-to-creatinine ratio is 20:1.

A 61 year-old woman who has had an upper respiratory infection for several weeks has presented to her family physician with complaints of a recent onset of urinary retention. She reveals to her physician that she has been taking non-prescription cold medications over and above the suggested dose for the past two weeks. Which of the following phenomena will her physician most likely suspect is contributing to her urinary retention?

The anticholinergic effects of the medication are impairing normal bladder function.

When explaining the role of liver Kupffer cells to a group of nursing students, which of the following statements about the function of these cells is most accurate?

The cells are capable of removing and phagocytizing old and defective blood cells.

An older adult states that he awakens at least three times each night to void. When assessing the client, what potential causative factor should the nurse prioritize?

The client takes his prescribed beta blocker and diuretic each evening at bedtime.

A busy 45-year-old female executive has been diagnosed with diverticulitis. Her primary treatment is an increase in the fibre content of her diet. What effect will the fibre have on the diverticula?

The fibre increases bulk, promotes regular defecation, and increases colonic contents and colon diameter, thereby decreasing intraluminal pressure.

A client suffering from chronic hypertension is beginning to show the symptoms of glomerular disease. This client's kidney damage is due to what phenomenon?

The higher pressures force protein and cells through the glomerular membrane, resulting in glomerular inflammation.

A 51 year-old male has been diagnosed with alcohol-induced liver disease. He admits to the nurse he does not understand what the liver does in the body. Which of these statements best explains the liver's function?

The liver metabolizes most components of food and also cleans the blood of bacteria and drugs.

A 56-year-old woman has been diagnosed with CKD. She first went to the doctor due to complications of hypertension. How are hypertension and CKD related?

The mechanisms that produce hypertension in CKD include an increased vascular volume and increased activity of the renin--angiotensin system.

During male ejaculation, which of the following statements addresses why sperm is not normally seen inside the bladder?

The musculature of the trigone area, bladder neck, and prostatic urethra contract at the same time.

When educating the public about hepatitis A, the nurse will stress which accurate teaching points? Select all that apply.

The onset of symptoms include high fever, malaise, abdominal pain, and jaundice. Persons working in various food industries like in a restaurant can pass on hepatitis A if they do not properly wash their hands after toilet. Be careful if traveling to a foreign country since many countries do not require vaccination of their workers.

Which of the following patients should have a feeling of bladder fullness?

The patient with 200 mL of urine in their bladder

A nurse observes that a patient's urine is cola colored and considers which of the following as a possible reason?

The patient's urine contains material from the degradation of red blood cells.

Urinary obstruction in the lower urinary tract triggers changes to the urinary system to compensate for the obstruction. What is an early change the system makes in its effort to cope with an obstruction?

The stretch receptors in the bladder wall become hypersensitive.

A number of clients on a geriatric subacute medical floor of a hospital have developed foul-smelling diarrhea over the last several days, and subsequent culture of stool samples has confirmed the presence of Clostridium difficile in each case. The care team on the unit would recognize that which of the following factors likely contributed to the health problem and would anticipate which of the following treatments?

The use of broad spectrum antibiotics likely played a role in the developments of the infections, and most clients would likely receive metronidazole as a treatment.

A young woman presents with signs and symptoms of urinary tract infection (UTI). The nurse notes that this is the fifth UTI in as many months. What would this information lead the nurse to believe?

There is possible obstruction in the urinary tract.

A physician's orders for a patient to be started on total parenteral nutrition (TPN). The nurse knows that TPN is a hypertonic solution and must be administered in which of the following ways?

Through a large diameter central line

The nurse is conducting a health education program on bladder cancer. The nurse teaches that which of the following is the most common type of bladder cancer?

Transitional carcinoma

Bruising of the skin is a manifestation of bleeding disorders in patients with CKD.

True

Constipation is the infrequent or difficult passage of stools.

True

Malabsorption syndrome results in loss of fat in the stool and failure to absorb fat-soluble vitamins.

True

The nursing student learns in her anatomy and physiology class that the bladder has how many main components?

Two

When determining the treatment and management for incontinence, the physician considers which of the following? Select all that apply.

Type of incontinence Accompanying health problems Person's age

The family members of a client who has been diagnosed with autosomal dominant polycystic kidney disease (ADPKD) asks the nurse what they should do to find out if they have this disease. The nurse will respond that they should visit with their healthcare provider and ask about having which lab/diagnostic test to screen for the disease?

Ultrasound of the kidney

The nursing student, while studying anatomy and physiology, correctly identifies which of the following to be responsible for carrying urine to the bladder?

Ureters

A client tells the nurse that he is experiencing involuntary loss of urine associated with a strong desire to void (urgency). The nurse would recognize this as:

Urge incontinence

The nurse understands that medications although very beneficial to clients can also have harmful effects. When working with elderly clients the nurse should recognize which of the following is a common result of potent, fast-acting diuretics?

Urge incontinence

A patient is being treated with colchicine (Colcrys) for pain in the big right toe. The patient begins to complain of severe right flank pain and is diagnosed with kidney stones. Which of the following types of kidney stones does the nurse recognize this patient is most likely affected by?

Uric acid

A patient is admitted with lower urinary tract obstruction and stasis. Which of the following is the primary intervention?

Urinary catheterization

Which of the following are risk factors for developing a UTI? (Select all that apply.)

Urinary obstruction Postmenopausal status Prostate disease Neurogenic bladder

Which clients have an increased risk for developing UTIs? A client who is or is diagnosed with: Select all that apply.

Urinary obstruction • Neurogenic disorders • Elderly • Prostate disease

A client is admitted to the emergency department with early symptoms of appendicitis. The nurse would assess the client for:

Vague pain that is referred to the epigastric or periumbilical area

Because the associated nephropathy is an important cause of end-stage renal failure in children and adolescents, a toddler who has had an uncomplicated bout of urinary tract infection (UTI) should still be evaluated for:

Vesicoureteral reflux

A teenager has been admitted with the diagnosis of Alport Syndrome. The client has been passing large amounts of blood in the urine. Knowing that this disorder is inherited and associated with other abnormalities, the nurse should focus assessment on which area?

Visual disturbances and hearing loss

The nurse walks into a room and finds the patient forcefully expelling stomach contents into a wash basin. When documenting this occurrence, the nurse will use the term:

Vomiting.

Which of the following clinical manifestations would lead the nurse to suspect their renal failure patient is developing uremia? Select all that apply.

Weakness and fatigue. Lethargy and confusion. Extreme itching.

Which of the following clinical manifestations would tell a nurse that a patient is having progressive decompensation related to obstruction of urinary outflow?

When tested for residual urine volume, 1400 mL of urine is obtained when patient is catheterized.

Select the option that best describes the sensory process involved in the inhibition of micturition.

When the bladder is distended to 150 to 250 mL in the adult, the sensation of fullness is transmitted to the spinal cord and then to the cerebral cortex, allowing for conscious inhibition of the micturition reflex.

An instructor is assisting a nursing student with inserting an indwelling catheter for a patient with urinary retention and acute overdistention of the bladder. The student inserts the catheter and gets an immediate return of clear yellow urine. When should the student clamp the catheter?

When the patient returns 1000 mL of urine from the bladder at once

A cytotechnologist is performing genetic testing on a series of tissues. One tissue comes back with the WT1 mutation, and it's mapped to chromosome 11. What disease will the client most likely develop?

Wilms tumour

When assessing the client during the icteric phase of viral hepatitis, which of these findings does the nurse anticipate observing?

Yellow-tinged sclera

The nurse is providing care for a 16-year-old male client, newly diagnosed with a spinal cord injury. He asks why he can no longer control his bladder. What would the nurse explain to him? (Select all that apply.)

Your spinal cord injury has disrupted the control your brain has over your bladder. • You will have to learn how to in-and-out catheterize yourself. • You have a condition known as detrusor-sphincter dyssynergia.

A rare condition caused by gastrin-secreting tumors most commonly found in the small intestine or pancreas is called which of the following?

Zollinger-Ellison syndrome

A child has been brought to an urgent care clinic. The parents state that the child is "not making water." When taking a history, the nurse learns the child had a sore throat about 1 week ago but seems to have gotten over it. "We [parents] only had to give antibiotics for 3 days for the throat to be better." The nurse should suspect the child has developed:

acute postinfectious glomerulonephritis.

A middle-aged man with diabetes reports that he must strain to urinate and that his urine stream is weak and dribbling. He also reports feeling that his bladder never really empties. His problem is most likely:

bladder atony and dysfunction arising from peripheral neuropathy.

The family asks the nurse what the usual treatment of focal segmental glomerulosclerosis entails. The nurse responds:

corticosteroids.

A 65-year-old Caucasian female who smokes presents with increased urinary frequency, dysuria, and sporadic, painless hematuria. The client's follow-up will likely include.

cystoscopy.

A client with chronic kidney disease has been admitted with increased shortness of breath and abnormal breath sounds (rales heard to scapular region of posterior back). The admission hemoglobin level is 7.8 g/dL. Vital signs are as follows: respiratory rate 30; BP 180/98; pulse 110. +3 pitting edema in lower extremities bilaterally. Knowing the correlation of hypertension and associated anemia, the nurse suspects this client has developed:

heart failure.

Which situations would result in an increased intestinal production of ammonia? A client who: Select all that apply.

ingested a high-protein lunch. was admitted with a gastrointestinal (GI) bleed.

A 40 year-old man who uses heroin intravenously was diagnosed with hepatitis C (HCV) one year ago and now has chronic viral hepatitis. Which of the following statements by the client to his care provider would warrant correction by the nurse?

it is at least a bit reassuring that my liver isn't undergoing damage when I'm not experiencing symptoms.

A child has been admitted to the pediatric unit with a tentative diagnosis of lipoid nephrosis. While taking the history, which items lead the nurse to suspect the diagnosis is accurate? The child has a history of: Select all that apply.

recent upper respiratory infection. recent routine vaccines.

A 68-year-old man who has smoked for at least 50 years reports that lately he feels as though food is "getting stuck" in his throat. At first this was a problem just with dry food, but now his morning oatmeal is getting stuck. On questioning, he reports drinking at least five alcoholic beverages nearly every day. His problem is most likely:

squamous cell carcinoma of the esophagus.

A client has been diagnosed with choledocholithiasis and acute suppurative cholangitis, which have been determined to be a surgical emergency. Surgery must take place promptly because:

the accumulation of purulent bile can cause sepsis.

During a physical assessment of a newborn in the nursery, the nurse palpates a mass over the kidney area. Testing reveals the infant has multicystic dysplastic kidney disorder. While educating the family, the nurse will share that many times the cysts will regress without intervention. However, the parents need to know the child will need annual: Select all that apply.

ultrasound, assessing cysts for possible cancer formation. blood pressure measurement, assuring proper functioning of unaffected kidney

The nurse recognizes that which of the following individuals have the greatest risk for the development of chronic kidney disease (CKD)? Select all that apply.

• A patient with a recent diagnosis of type 2 diabetes who does not monitor his blood sugar or control his diet • A 42-year-old patient with uncontrolled hypertension

The nurse knows that a patient with chronic kidney disease (CKD) may develop renal osteodystrophy (skeletal damage). Which of the following are characteristic of both high-bone-turnover and low-bone-turnover osteodystrophy? Select all that apply.

• Abnormal bone resorption • Defective bone remodeling

A client who is suspected of having acute renal failure would first be assessed by which of the following blood tests? Select all that apply.

• Blood urea nitrogen • Creatinine

A patient has developed chronic kidney disease (CKD). The nurse will advise the patient to be alert for the development of which hematologic signs and symptoms of this disorder? Select all that apply.

• Bruising • Gastrointestinal bleeding • Fatigue

A client is diagnosed with decreased gomerular filtration rate but has no renal damage. The nurse recognizes that this can occur with which of the following? Select all that apply.

• Cirrhosis • Heart failure • Removal of one kidney • Dehydration

Anemia frequently occurs in clients with chronic kidney disease (CKD). The nurse will monitor these clients for which of the following contributors to anemia? Select all that apply.

• Decreased erythropoietin production • Anorexia • Bone marrow suppression • Chronic blood loss

A patient is diagnosed with chronic kidney disease (CKD). The nurse recognizes that this patient will experience which of the following? Select all that apply.

• Decreased renal endocrine function • Decreased tubular reabsorption • Decreased glomerular filtration

When conducting an admission interview with a client with a history of urinary incontinence, the nurse will specifically ask whether the client is prescribed which classification of medications in order to determine a possible cause? Select all that apply.

• Diuretics • Hypnotics • Sedatives

In a client in the uremic state of chronic kidney failure (CKD), the nurse will monitor for which of the following signs and symptoms? Select all that apply.

• Encephalopathy • Nausea • Anemia • Apathy

Glomerular filtration rate (GFR) is the best indicator of renal function. The nurse knows that GFR can vary with which of the following factors? Select all that apply.

• Gender • Age • Body size • Ethnicity

A client with ascites suddenly develops a change in mental status, elevated neutrophil count, and abdominal pain. For what additional complications will the nurse monitor the client? Select all that apply.

• Hepatic encephalopathy • Shock • Diarrhea

Which diagnosis causes an increased risk of developing intrahepatic jaundice? Select all that apply.

• Hepatitis • Cirrhosis • Liver cancer

Chronic kidney disease (CKD) can cause derangement of calcium and phosphate metabolism. The nurse will monitor a client with CKD for which of the following? Select all that apply.

• Impaired phosphate excretion • Decreased blood levels of calcium

A client is diagnosed with chronic kidney disease (CKD). The nurse will monitor for which of the following manifestations of this disorder? Select all that apply.

• Increased nitrogenous waste in blood • Anemia • Osteodystrophy

A client is diagnosed with acute renal injury. The nurse will evaluate the client for which of the following possible causes for this disorder? Select all that apply.

• Ischemic injury • Nonischemic reduced renal blood flow • Obstruction of urinary outflow • Nephrotoxic substances

A nurse is caring for a patient diagnosed with nephrotic syndrome. The nurse is aware that the patient may manifest which of the following? Select all that apply.

• Massive proteinuria (>3.5 g/day) • Lipiduria • Generalized edema • Hyperlipidemia

The nurse is caring for a group of patients in the acute care unit. Which of the following patients are at risk for developing intrarenal failure? Select all that apply.

• One who is taking gentamicin for the treatment of a kidney infection • One with lead poisoning • One with acute glomerulonephritis

The nurse knows that a patient with chronic kidney disease (CKD) may experience which of the following changes in skin integrity? Select all that apply.

• Pale skin • Brittle fingernails • Decreased perspiration

Which of the following patients does the nurse need to monitor for the development of intense intrarenal vasoconstriction that may induce prerenal failure? Select all that apply.

• Patient undergoing cardiac catheterization • Patient taking cyclosporine to prevent rejection for a liver transplant • Patient with an elevated blood urea nitrogen who is taking ibuprofen every 6 hours for back pain

A client with CKD is being treated for hyperphosphatemia and hypocalcemia. Select the most appropriate intervention. Select all that apply.

• Phosphate-binding antacids • Activated vitamin D • Restriction of foods high in phosphate

A nurse is caring for a child with Wilms tumor, stage I. Which of the following can be said regarding this diagnosis? Select all that apply.

• Prognosis is good with treatment. • The tumor is limited to the kidney and can be excised with the capsular surface intact.

A patient has developed acute tubular necrosis (ATN). The nurse knows that which of the following groups of drugs can cause this type of renal injury? Select all that apply.

• Radiocontrast dyes • Chemotherapy drugs • Aminoglycoside anti-infectives

Many drugs and other nephrotoxic agents can induce nephrotic acute tubular necrosis (ATN). The nurse knows that these agents cause tubular injury by which of the following mechanisms? Select all that apply.

• Renal vasoconstriction • Intratubular obstruction • Direct tubular damage

A client is diagnosed with early chronic kidney disease (CKD). The nurse will recommend which of the following actions to slow progression of renal damage? Select all that apply.

• Smoking cessation • Blood pressure control • Angiotensin converting enzyme inhibitor (ACEI) administration • Blood glucose control

The nursing students have learned in class that causes of urinary obstruction and urinary incontinence include which of the following? Select all that apply.

• Structural changes in the bladder • Structural changes in the urethra • Impairment of neurologic control of bladder function

The nurse recognizes that renal failure has many underlying causes, including which of the following? Select all that apply

• Systemic disease • Renal disease • Nonrenal urological abnormalities

The nurse will monitor clients with which of the following disorders for development of chronic kidney disease (CKD)? Select all that apply.

• Systemic lupus erythematosus • Polycycstic kidney disease • Glomerulonephritis • Diabetes

A client has been admitted with suspected kidney cancer. The client has had blood in the urine and some flank pain, but associated these sympstoms with a kidney stone, because of a previous experience This episode didn't go away, so the client sought treatment. The client asks the nurse what testing procedures would be ordered to confirm the diagnosis? The nurse responds: Select all that apply.

• ultrasound of the kidney looking for masses. • computerized tomography (CT) scan looking for lesions and metastasis. • magnetic resonance imaging (MRI) to identify if the inferior vena cava is involved


Kaugnay na mga set ng pag-aaral

Anatomy Exam 4 - Proximal & Distal Attachments, Innervation, Action

View Set

1.3 Culture EAQ, Culture adaptive quiz, Adaptive quiz: professional identity, 38 Case Study, section 2 1st semester

View Set

Chapter Quiz EXAM FX (Completing the applications, Underwriting, and Delivering the policy.

View Set